2020 AMC 10A Solution
2020 AMC 10A Solution
Problem1
Solution
Adding to both
sides, .
Problem2
Solution
to . Solving for ,
Solution
Note that is times .
Likewise, is times and is times .
Therefore, the product of the given fraction
equals .
Problem4
A driver travels for hours at miles per hour, during which her car
gets miles per gallon of gasoline. She is paid per mile, and her only
expense is gasoline at per gallon. What is her net rate of pay, in dollars
per hour, after this expense?
Solution
Since the driver travels 60 miles per hour and each hour she uses 2 gallons of
gasoline, she spends $4 per hour on gas. If she gets $0.50 per mile, then she
gets $30 per hour of driving. Subtracting the gas cost, her net rate of pay per
hour is .
Problem5
Case 1:
Case 2:
Similarly, taking the nonpositive case for the value inside the absolute value
Solution 2
We have the
equations and .
Notice that the second is a perfect square with a double root at , and the
first has real roots. By Vieta's, the sum of the roots of the first equation
is .
Problem6
How many -digit positive integers (that is, integers between and ,
inclusive) having only even digits are divisible by
Solution
The ones digit, for all numbers divisible by 5, must be either or . However,
from the restriction in the problem, it must be even, giving us exactly one choice
( ) for this digit. For the middle two digits, we may choose any even integer
from , meaning that we have total options. For the first digit, we follow
similar intuition but realize that it cannot be , hence giving us 4 possibilities.
Therefore, using the multiplication rule, we
get . ~ciceronii
Problem7
Solution
Without loss of generality, consider the five rows in the square. Each row must
have the same sum of numbers, meaning that the sum of all the numbers in the
square divided by is the total value per row. The sum of the integers
is , and the
common sum is .
Solution 2
Take the sum of the middle 5 values of the set (they will turn out to be the mean
Problem8
What is the value of
Solution 1
Split the even numbers and the odd numbers apart. If we group every 2 even
of .
Solution 2 (bashy)
We can break this entire sum down into integer bits, in which the sum is ,
where is the first integer in this bit. We can find that the first sum of every
sequence is , which we plug in for the bits in the entire sequence
is .-middletonkids
Solution 3
Another solution involves adding everything and subtracting out what is not
needed. The first step involves
solving
answer is .
—
Solution 4
In this solution, we group every 4 terms. Our groups should
be: , ,
, ...
as . We add this to
get . ~Baolan
Solution 5
We can split up this long sum into groups of four integers. Finding the first few
is , or . ~Arctic_Bunny
Solution 3
Taking the average of the first and last terms, and , we have that the
mean of the set is . There are 5 values in each row, column or diagonal, so the
value of the common sum is , or . ~Arctic_Bunny, edited by
KINGLOGIC
Problem9
A single bench section at a school event can hold either adults or children.
When bench sections are connected end to end, an equal number of adults
and children seated together will occupy all the bench space. What is the least
possible positive integer value of
Solution
The least common multiple of and is . Therefore, there must
be adults and children. The total number of benches
is .
Solution 2
This is similar to Solution 1, with the same basic idea, but we don't need to
calculate the LCM. Since both and are prime, their LCM must be their
Problem10
Seven cubes, whose volumes are , , , , , , and cubic
units, are stacked vertically to form a tower in which the volumes of the cubes
decrease from bottom to top. Except for the bottom cube, the bottom face of each
cube lies completely on top of the cube below it. What is the total surface area of
the tower (including the bottom) in square units?
Solution 1
The volume of each cube follows the pattern of ascending, for is
between and .
We see that the total surface area can be comprised of three parts: the sides of
the cubes, the tops of the cubes, and the bottom of the cube
(which is just ). The sides areas can be measured as the
~ciceronii
Solution 2
It can quickly be seen that the side lengths of the cubes are the integers from 1 to
7, inclusive.
First, we will calculate the total surface area of the cubes, ignoring overlap. This
value
is
. Then, we need to subtract out the overlapped parts of the cubes. Between each
consecutive pair of cubes, one of the smaller cube's faces is completely covered,
along with an equal area of one of the larger cube's faces. The total area of the
get . ~emerald_block
.
We then come up with
this:
Problem 11
What is the median of the following list of numbers
Solution 1
and equal to . Since is equal to , it, with the other squares, will
shift our median's placement up . We can find that the median of the whole
set is , and gives us . Our answer
is .
~aryam
Solution 2
As we are trying to find the median of a -term set, we must find the
average of the th and st terms.
get ~emerald_block
Solution 3
We want to know the th term and the th term to get the median.
We know that
Problem12
Triangle is isoceles with .
so Thus,
Solution 2 (Trapezoid)
We know that , and since the ratios of its sides
Area of
Area of
is .
This is of the triangle, so the area of the triangle
Solution 3 (Medians)
Draw median .
Since we know that all medians of a triangle intersect at the incenter, we know
Finally knowing that the medians divide the triangle into sections of equal area,
us
~quacker88
Solution 4 (Triangles)
We know
that , , so .
As , we can see
that and with a side
ratio of .
So , .
As said in solution 1, .
triangle ,
so . Since is a median, .
From right
triangle ,
which implies . By
symmetry .
Applying the Pythagorean Theorem to right
triangle gives
Solution 7
Given a triangle with perpendicular medians with lengths and , the area will
be .
Solution 8 (Fastest)
Connect the line segment and it's easy to see
quadrilateral has an area of the product of its diagonals divided
by which is . Now, solving for triangle could be an option, but the
drawing shows the area of will be less than the quadrilateral meaning
the the area of is less than but greater than , leaving only
Problem 13
A frog sitting at the point begins a sequence of jumps, where each jump
is parallel to one of the coordinate axes and has length , and the direction of
each jump (up, down, right, or left) is chosen independently at random. The
sequence ends when the frog reaches a side of the square with
Solution
Drawing out the square, it's easy to see that if the frog goes to the left, it will
immediately hit a vertical end of the square. Therefore, the probability of this
sides of the square), the chance it will hit a vertical side of a square is . The
If the frog goes either up or down, it will hit a line of symmetry along the corner it
is closest to and furthest to, and again, is equidistant relating to the two closer
sides and also equidistant relating the two further sides. The probability for it to
hit a vertical wall is . Because there's a chance of the frog going up and
down, the total probability for this case is and summing up all the
cases,
Solution 2
Let's say we have our four by four grid and we work this out by casework. A is
where the frog is, while B and C are possible locations for his second jump, while
O is everything else. If we land on a C, we have reached the vertical side.
However, if we land on a B, we can see that there is an equal chance of reaching
the horizontal or vertical side, since we are symmetrically between them. So we
have the probability of landing on a C is 1/4, while B is 3/4. Since C means that
we have "succeeded", while B means that we have a half chance, we
compute .
We get , or
-yeskay
Solution 3
If the frog is on one of the 2 diagonals, the chance of landing on vertical or
down, or right) it will reach a diagonal on the first jump and chance (left) it will
reach the vertical side. The probablity of landing on a vertical
is - Lingjun.
Let denote the probability of the frog's sequence of jumps ends with it
Similarly, , and .
Now we create equations for the probabilities at each of these points/states by
considering the probability of going either up, down, left, or right from that
point:
gives
Plugging in the third equation into this
gives
Next, plugging
in the second and third equation into the first equation
yields
get
Problem14
value of
Solution
Continuing to
combine
into , we
have . ~PCChess
Solution 2
so and .
Thus
answer is .
Solution 3
us Recalling
get
get ~emerald_block
Solution 4 (Bashing)
This is basically bashing using Vieta's formulas to find and (which I highly do
not recommend, I only wrote this solution for fun).
reduce to
would be .
expansions and will cancel out (since a positive plus a negative of the
same absolute value equals zero). We also observe that the other terms not
Problem15
A positive integer divisor of is chosen at random. The probability that the
Solution
Solution 1
Diagram
Solution
We consider an individual one-by-one block.
If we draw a quarter of a circle from each corner (where the lattice points are
located), each with radius , the area covered by the circles should be .
Because of this, and the fact that there are four circles, we write
that ~Crypthes
clearly the probability is greater than . This would make sure the above solution
Solution 2
so .
Problem 17
Define How many
Solution 1
Solution 2
Notice that is nonpositive when is
.
This reduces
to
~Zeric
is, .
Since the degree of is even, its end behaviors match. And since the
leading coefficient is positive, we know that both ends approach as goes in
either direction.
an -intercept and it's going to dip below. This happens at , which is the
smallest intercept.
However, when it hits the next intercept, it's going to go back up again into
positive territory, we know this happens at . And when it hits , it's going to
dip back into negative territory. Clearly, this is going to continue to snake around
To get the amount of integers below and/or on the -axis, we simply need to
count the integers. For example, the amount of integers in between
.
Proceed with Solution 2. ~quacker88
Problem18
each one of them in the set For how many such quadruples is it true
Solution
Solution 1 (Parity)
In order for to be odd, consider parity. We must have (even)-
(odd) or (odd)-(even). There are ways to pick numbers
to obtain an even product. There are ways to obtain an odd product.
Therefore, the total amount of ways to make odd
is .
-Midnight
count: ,
which is . The number of ways to get an odd product can
Problem19
As shown in the figure below, a regular dodecahedron (the polyhedron consisting
of congruent regular pentagonal faces) floats in space with two horizontal
faces. Note that there is a ring of five slanted faces adjacent to the top face, and
a ring of five slanted faces adjacent to the bottom face. How many ways are there
to move from the top face to the bottom face via a sequence of adjacent faces so
that each face is visited at most once and moves are not permitted from the
bottom ring to the top ring?
Diagram
Solution 1
Since we start at the top face and end at the bottom face without moving from the
lower ring to the upper ring or revisiting a face, our journey must consist of the
top face, a series of faces in the upper ring, a series of faces in the lower ring,
and the bottom face, in that order.
We have choices for which face we visit first on the top ring. From there, we
have choices for how far around the top ring we go before moving
get .
Solution 2
Swap the faces as vertices and the vertices as faces. Then, this problem is the
same as 2016 AIME I #3which had an answer
of .
Problem20
Quadrilateral satisfies
and Diagona
need to find to get the area of the whole quadrilateral. Drop an altitude
from to and call the point of intersection . Let .
Since , then . By dropping this altitude, we can also
see two similar triangles, and .
Since is , and , we get that .
Now, if we redraw another diagram just of , we get
that . Now expanding, simplifying, and
Since , .
So
(I'm very sorry if you're a visual learner but now you have a diagram by ciceronii)
~ Solution by Ultraman
~ Diagram by ciceronii
legs is a multiple of 20. Guess that the legs are equal to and ,
Solution 3 (coordinates)
Let the points
be , , , ,and ,
Solution 4 (Trigonometry)
Let and Using Law of Sines
Divide the
two to
get Now,
and
solve the quadratic, taking the positive solution (C is acute) to
get So
is
(This solution is incomplete, can someone complete it please-Lingjun) ok Latex
edited by kc5170
We could use the famous m-n rule in trigonometry in triangle ABC with Point E
[Unable to write it here.Could anybody write the expression] We will find that BD
is angle bisector of triangle ABC(because we will get tan (x)=1) Therefore by
converse of angle bisector theorem AB:BC = 1:3. By using phythagorean
theorem we have values of AB and AC. AB.AC = 120. Adding area of ABC and
ACD Answer••360
Problem21
There exists a unique strictly increasing sequence of nonnegative
integers such
that What is
Solution 1
becomes . Now
equals . Note
is .
~seanyoon777
Solution 2
answer is .
Solution 3 (Intuitive)
Solution 4
Note that the expression is equal to something slightly lower than . Clearly,
apart, but because the expression is odd, it will have to contain , and
Solution 5
Note that
and
Since
so
Expressing each of the pairs of the form in binary, we have
or
Accounting for the term, which was not in the pair, we have a total
of terms.
Problem22
For how many positive
Solution 1 (Casework)
Expression:
Solution:
Let
the largest and smallest terms before the function is applied is less than or
equal to , and thus the terms must have a range of or less after the function is
applied.
above must be ,
be ,
be ,
be , and
which is divisible by 3.
Case 3: divides
Because divides , the number of possibilities for is the same as the
number of factors of .
Case 4: divides
Because divides , the number of possibilities for is the same as the
number of factors of .
Similar to Case 1, the value of the terms of the expression are . The
sum is , which is divisible by 3, so this case does not work (0 cases).
, so the answer is .
~dragonchomper, additional edits by emerald_block
Notice that you only need to count the number of factors of 1000 and 999,
excluding 1. 1000 has 16 factors, and 999 has 8. Adding them gives you 24, but
you need to subtract 2 since 1 does not work.
Solution 3
NOTE: For this problem, whenever I say , I will be referring to all the
factors of the number except for .
Now, quickly observe that if divides ,
logic, we can deduce that too works (for our problem). Thus,
we need the factors of and and we don't have to eliminate any
Problem23
Let be the triangle in the coordinate plane with
Solution
First, any combination of motions we can make must reflect an even number
of times. This is because every time we reflect , it changes orientation.
Once has been flipped once, no combination of rotations will put it back in
place because it is the mirror image; however, flipping it again changes it back to
the original orientation. Since we are only allowed transformations and an even
number of them must be reflections, we either reflect times or times.
Case 1: 0 reflections on T
In this case, we must use rotations to return to its original position. Notice
That way, the composition of rotations yields a full rotation. For example,
if ,
then ,
The only case in which this fails is when would have to equal . This happens
namely, or .
However, we can simply subtract these three cases from the total.
Case 2: 2 reflections on T
In this case, we first eliminate the possibility of having two of the same reflection.
Since two reflections across the x-axis maps back to itself, inserting a rotation
before, between, or after these two reflections would change 's final location,
meaning that any combination involving two reflections across the x-axis would
not map back to itself. The same applies to two reflections across the y-axis.
Therefore, we must use one reflection about the x-axis, one reflection about the
y-axis, and one rotation. Since a reflection about the x-axis changes the sign of
the y component, a reflection about the y-axis changes the sign of the x
component, and a rotation changes both signs, these three transformation
composed (in any order) will suffice. It is therefore only a question of arranging
the three, giving us combinations for case 2.
Combining both cases we get
in ways.
Suppose there are two reflections. As noted already, they must be different, and
as a result will take the triangle to the opposite side of the origin if we don't do
any rotation. We have 1 rotation left that we can do though, and the only one that
will return to the original position is 2, which is AKA reflection across
origin. Therefore, since all 3 transformations are distinct. The three
transformations can be applied anywhere since they are commutative(think
quadrants). This gives ways.
Problem24
Let be the least positive integer greater than for
which
Solution 1
We know that , so we can
write . Simplifying, we
write , or . Solving
where or .
so we
try . s
o again we add to . It turns out
Solution 2 (bashing)
We are given
that and .
that is divisible by 60 but not 120. Starting, we find the least value
problem, and .
So we know that (last digit of ). That means
that and . We
can bash this after this. We just want to find all pairs of numbers such
that is a multiple of 7 that is greater than a multiple of . Our equation
Solution 4
The conditions of the problem reduce to the
Solution 5
You can first find that n must be congruent
= .-happykeeper
that and By
applying the Euclidean algorithm, but in reverse, we
have
and
integer We know that or else the first condition won't hold ( will
is
Problem25
Jason rolls three fair standard six-sided dice. Then he looks at the rolls and
chooses a subset of the dice (possibly empty, possibly all three dice) to reroll.
After rerolling, he wins if and only if the sum of the numbers face up on the three
dice is exactly Jason always plays to optimize his chances of winning. What is
the probability that he chooses to reroll exactly two of the dice?
Solution 1
Consider the probability that rolling two dice gives a sum of , where .
There are pairs that satisfy this,
namely , out
Therefore, if one die has a value of and Jason rerolls the other two dice, then
Thus, we can let be the values of the three dice, which we will
the sum of the three dice . This happens with probability . If we reroll two
However, , so rolling one die is always better than rolling two dice
if .
Now consider the case where . Rerolling one die will not help us
win since the sum of the three dice will always be greater than . If we reroll two
dice, the probability of winning is, once again, . To find the probability of
winning if we reroll all three dice, we can let each dice have dot and find the
number of ways to distribute the remaining dots. By Stars and Bars, there
winning .
In order for rolling two dice to be more favorable than rolling three
dice, .
possible triplets that satisfy these conditions, and the number of ways
they can be permuted,
ways.
Solution 2
We count the numerator. Jason will pick up no dice if he already has a 7 as a
sum. We need to assume he does not have a 7 to begin with. If Jason decides to
pick up all the dice to re-roll, by Stars and Bars(or whatever...), there will be 2
bars and 4 stars(3 of them need to be guaranteed because a roll is at least 1) for
7, since this will give him a chance which is a larger probability than all the
cases unless he has a 7 to begin with. We
have